Alternating Series Doubts












0












$begingroup$


I've some doubts on Alternating series and Alternating test series. I was trying to clear my mind by practicing with some of my book exercises and found troubles with this one:



$$sum_{n=0}^infty frac{(-1)^{n+1}}{sqrt[n+1] {10}}$$



My logic is as follows:



The series doesn't converge since my $a_n = - frac{1}{sqrt[n+1]{10}}$ approaches $1$ as $n$ goes to $infty$ and is $decreasing$ because of $sqrt[n+1] {10} > sqrt[n+2] {10}$



I don't want to stop here so my question:
is there a way to know if this series diverges to $+infty$ or $-infty$?










share|cite|improve this question











$endgroup$












  • $begingroup$
    Neither. It oscillates.
    $endgroup$
    – Xander Henderson
    Dec 1 '18 at 17:41










  • $begingroup$
    @XanderHenderson everytime i will be in this situation the series will be oscillating? if my $a_n$ approaches $0$ and is not $decreasing$ the series is also oscillating?
    $endgroup$
    – Lorenzo Cavallini
    Dec 1 '18 at 17:47








  • 1




    $begingroup$
    Every? That is a rather broad blanket statement... It will depend on your specific situation.
    $endgroup$
    – JMoravitz
    Dec 1 '18 at 17:59










  • $begingroup$
    please, can you make an example of alternating series which diverges to $+infty or -infty$ for sure?
    $endgroup$
    – Lorenzo Cavallini
    Dec 1 '18 at 18:44
















0












$begingroup$


I've some doubts on Alternating series and Alternating test series. I was trying to clear my mind by practicing with some of my book exercises and found troubles with this one:



$$sum_{n=0}^infty frac{(-1)^{n+1}}{sqrt[n+1] {10}}$$



My logic is as follows:



The series doesn't converge since my $a_n = - frac{1}{sqrt[n+1]{10}}$ approaches $1$ as $n$ goes to $infty$ and is $decreasing$ because of $sqrt[n+1] {10} > sqrt[n+2] {10}$



I don't want to stop here so my question:
is there a way to know if this series diverges to $+infty$ or $-infty$?










share|cite|improve this question











$endgroup$












  • $begingroup$
    Neither. It oscillates.
    $endgroup$
    – Xander Henderson
    Dec 1 '18 at 17:41










  • $begingroup$
    @XanderHenderson everytime i will be in this situation the series will be oscillating? if my $a_n$ approaches $0$ and is not $decreasing$ the series is also oscillating?
    $endgroup$
    – Lorenzo Cavallini
    Dec 1 '18 at 17:47








  • 1




    $begingroup$
    Every? That is a rather broad blanket statement... It will depend on your specific situation.
    $endgroup$
    – JMoravitz
    Dec 1 '18 at 17:59










  • $begingroup$
    please, can you make an example of alternating series which diverges to $+infty or -infty$ for sure?
    $endgroup$
    – Lorenzo Cavallini
    Dec 1 '18 at 18:44














0












0








0





$begingroup$


I've some doubts on Alternating series and Alternating test series. I was trying to clear my mind by practicing with some of my book exercises and found troubles with this one:



$$sum_{n=0}^infty frac{(-1)^{n+1}}{sqrt[n+1] {10}}$$



My logic is as follows:



The series doesn't converge since my $a_n = - frac{1}{sqrt[n+1]{10}}$ approaches $1$ as $n$ goes to $infty$ and is $decreasing$ because of $sqrt[n+1] {10} > sqrt[n+2] {10}$



I don't want to stop here so my question:
is there a way to know if this series diverges to $+infty$ or $-infty$?










share|cite|improve this question











$endgroup$




I've some doubts on Alternating series and Alternating test series. I was trying to clear my mind by practicing with some of my book exercises and found troubles with this one:



$$sum_{n=0}^infty frac{(-1)^{n+1}}{sqrt[n+1] {10}}$$



My logic is as follows:



The series doesn't converge since my $a_n = - frac{1}{sqrt[n+1]{10}}$ approaches $1$ as $n$ goes to $infty$ and is $decreasing$ because of $sqrt[n+1] {10} > sqrt[n+2] {10}$



I don't want to stop here so my question:
is there a way to know if this series diverges to $+infty$ or $-infty$?







sequences-and-series summation divergent-series






share|cite|improve this question















share|cite|improve this question













share|cite|improve this question




share|cite|improve this question








edited Dec 1 '18 at 17:39









Xander Henderson

14.3k103554




14.3k103554










asked Dec 1 '18 at 17:37









Lorenzo CavalliniLorenzo Cavallini

103




103












  • $begingroup$
    Neither. It oscillates.
    $endgroup$
    – Xander Henderson
    Dec 1 '18 at 17:41










  • $begingroup$
    @XanderHenderson everytime i will be in this situation the series will be oscillating? if my $a_n$ approaches $0$ and is not $decreasing$ the series is also oscillating?
    $endgroup$
    – Lorenzo Cavallini
    Dec 1 '18 at 17:47








  • 1




    $begingroup$
    Every? That is a rather broad blanket statement... It will depend on your specific situation.
    $endgroup$
    – JMoravitz
    Dec 1 '18 at 17:59










  • $begingroup$
    please, can you make an example of alternating series which diverges to $+infty or -infty$ for sure?
    $endgroup$
    – Lorenzo Cavallini
    Dec 1 '18 at 18:44


















  • $begingroup$
    Neither. It oscillates.
    $endgroup$
    – Xander Henderson
    Dec 1 '18 at 17:41










  • $begingroup$
    @XanderHenderson everytime i will be in this situation the series will be oscillating? if my $a_n$ approaches $0$ and is not $decreasing$ the series is also oscillating?
    $endgroup$
    – Lorenzo Cavallini
    Dec 1 '18 at 17:47








  • 1




    $begingroup$
    Every? That is a rather broad blanket statement... It will depend on your specific situation.
    $endgroup$
    – JMoravitz
    Dec 1 '18 at 17:59










  • $begingroup$
    please, can you make an example of alternating series which diverges to $+infty or -infty$ for sure?
    $endgroup$
    – Lorenzo Cavallini
    Dec 1 '18 at 18:44
















$begingroup$
Neither. It oscillates.
$endgroup$
– Xander Henderson
Dec 1 '18 at 17:41




$begingroup$
Neither. It oscillates.
$endgroup$
– Xander Henderson
Dec 1 '18 at 17:41












$begingroup$
@XanderHenderson everytime i will be in this situation the series will be oscillating? if my $a_n$ approaches $0$ and is not $decreasing$ the series is also oscillating?
$endgroup$
– Lorenzo Cavallini
Dec 1 '18 at 17:47






$begingroup$
@XanderHenderson everytime i will be in this situation the series will be oscillating? if my $a_n$ approaches $0$ and is not $decreasing$ the series is also oscillating?
$endgroup$
– Lorenzo Cavallini
Dec 1 '18 at 17:47






1




1




$begingroup$
Every? That is a rather broad blanket statement... It will depend on your specific situation.
$endgroup$
– JMoravitz
Dec 1 '18 at 17:59




$begingroup$
Every? That is a rather broad blanket statement... It will depend on your specific situation.
$endgroup$
– JMoravitz
Dec 1 '18 at 17:59












$begingroup$
please, can you make an example of alternating series which diverges to $+infty or -infty$ for sure?
$endgroup$
– Lorenzo Cavallini
Dec 1 '18 at 18:44




$begingroup$
please, can you make an example of alternating series which diverges to $+infty or -infty$ for sure?
$endgroup$
– Lorenzo Cavallini
Dec 1 '18 at 18:44










1 Answer
1






active

oldest

votes


















0












$begingroup$

First, it is very important in mathematics to carefully read and understand the definitions that are in play. In this case, the following are probably relevant:




Definition: Let ${a_n}$ be a sequence of real numbers. We say that the series
$$ sum_{n=1}^{infty} a_n $$
converges if the sequence of partial sums
$$ S_N := sum_{n=1}^{N} a_n $$
converges. Oherwise, we say that the series diverges.




That is, understanding the behaviour of a series comes down to understanding the limiting behaviour of the sequence of partial sums. So, instead of asking about the series, let's step back for a second and ask how the sequence of partial sums might behave. If the series diverges, there are basically three possibilities:




  1. It could be that for any $M > 0$, there exists an $N$ large enough that $n > N$ implies that $S_n > M$. In this case, we say that the sequence of partial sums (and the corresponding series) diverge to $infty$. For example:
    $$ sum_{n=1}^{infty} 1 qquadtext{or}qquad sum_{n=1}^{infty} frac{1}{n}. $$

  2. It could be that for any $M > 0$, there exists an $N$ large enough that $n > N$ implies that $S_n < -M$. In this case, we say that the sequence of partial sums (and the corresponding series) diverge to $-infty$.
    $$ sum_{n=1}^{infty} frac{1}{log(frac{1}{n})}. $$

  3. It could be that neither of these things occurs. In this case, the sequence (and the corresponding series) simply diverge, and no other descriptors are necessary, though we might be able to describe the behaviour in a little more detail if we care to. For example, the partial sums could bounce back and forth between to specific values, or they could oscillate with increasingly large oscillations, or they fall into some cycle, or they could simply bounce around at random.


In the case of the series presented in the question, the alternating series test shows that the series diverges (as you point out), but this is about the most that we can say without using some other tools. Very roughly speaking, we can see that the difference between consecutive partial sums increases to 1, so the sizes of the oscillations are bounded. There is also enough symmetry in the series to imply that we are not consistently adding more than we are subtracting, hence it is reasonable to think that the sequence of partial sums oscillates, but is bounded.






share|cite|improve this answer











$endgroup$













    Your Answer





    StackExchange.ifUsing("editor", function () {
    return StackExchange.using("mathjaxEditing", function () {
    StackExchange.MarkdownEditor.creationCallbacks.add(function (editor, postfix) {
    StackExchange.mathjaxEditing.prepareWmdForMathJax(editor, postfix, [["$", "$"], ["\\(","\\)"]]);
    });
    });
    }, "mathjax-editing");

    StackExchange.ready(function() {
    var channelOptions = {
    tags: "".split(" "),
    id: "69"
    };
    initTagRenderer("".split(" "), "".split(" "), channelOptions);

    StackExchange.using("externalEditor", function() {
    // Have to fire editor after snippets, if snippets enabled
    if (StackExchange.settings.snippets.snippetsEnabled) {
    StackExchange.using("snippets", function() {
    createEditor();
    });
    }
    else {
    createEditor();
    }
    });

    function createEditor() {
    StackExchange.prepareEditor({
    heartbeatType: 'answer',
    autoActivateHeartbeat: false,
    convertImagesToLinks: true,
    noModals: true,
    showLowRepImageUploadWarning: true,
    reputationToPostImages: 10,
    bindNavPrevention: true,
    postfix: "",
    imageUploader: {
    brandingHtml: "Powered by u003ca class="icon-imgur-white" href="https://imgur.com/"u003eu003c/au003e",
    contentPolicyHtml: "User contributions licensed under u003ca href="https://creativecommons.org/licenses/by-sa/3.0/"u003ecc by-sa 3.0 with attribution requiredu003c/au003e u003ca href="https://stackoverflow.com/legal/content-policy"u003e(content policy)u003c/au003e",
    allowUrls: true
    },
    noCode: true, onDemand: true,
    discardSelector: ".discard-answer"
    ,immediatelyShowMarkdownHelp:true
    });


    }
    });














    draft saved

    draft discarded


















    StackExchange.ready(
    function () {
    StackExchange.openid.initPostLogin('.new-post-login', 'https%3a%2f%2fmath.stackexchange.com%2fquestions%2f3021606%2falternating-series-doubts%23new-answer', 'question_page');
    }
    );

    Post as a guest















    Required, but never shown

























    1 Answer
    1






    active

    oldest

    votes








    1 Answer
    1






    active

    oldest

    votes









    active

    oldest

    votes






    active

    oldest

    votes









    0












    $begingroup$

    First, it is very important in mathematics to carefully read and understand the definitions that are in play. In this case, the following are probably relevant:




    Definition: Let ${a_n}$ be a sequence of real numbers. We say that the series
    $$ sum_{n=1}^{infty} a_n $$
    converges if the sequence of partial sums
    $$ S_N := sum_{n=1}^{N} a_n $$
    converges. Oherwise, we say that the series diverges.




    That is, understanding the behaviour of a series comes down to understanding the limiting behaviour of the sequence of partial sums. So, instead of asking about the series, let's step back for a second and ask how the sequence of partial sums might behave. If the series diverges, there are basically three possibilities:




    1. It could be that for any $M > 0$, there exists an $N$ large enough that $n > N$ implies that $S_n > M$. In this case, we say that the sequence of partial sums (and the corresponding series) diverge to $infty$. For example:
      $$ sum_{n=1}^{infty} 1 qquadtext{or}qquad sum_{n=1}^{infty} frac{1}{n}. $$

    2. It could be that for any $M > 0$, there exists an $N$ large enough that $n > N$ implies that $S_n < -M$. In this case, we say that the sequence of partial sums (and the corresponding series) diverge to $-infty$.
      $$ sum_{n=1}^{infty} frac{1}{log(frac{1}{n})}. $$

    3. It could be that neither of these things occurs. In this case, the sequence (and the corresponding series) simply diverge, and no other descriptors are necessary, though we might be able to describe the behaviour in a little more detail if we care to. For example, the partial sums could bounce back and forth between to specific values, or they could oscillate with increasingly large oscillations, or they fall into some cycle, or they could simply bounce around at random.


    In the case of the series presented in the question, the alternating series test shows that the series diverges (as you point out), but this is about the most that we can say without using some other tools. Very roughly speaking, we can see that the difference between consecutive partial sums increases to 1, so the sizes of the oscillations are bounded. There is also enough symmetry in the series to imply that we are not consistently adding more than we are subtracting, hence it is reasonable to think that the sequence of partial sums oscillates, but is bounded.






    share|cite|improve this answer











    $endgroup$


















      0












      $begingroup$

      First, it is very important in mathematics to carefully read and understand the definitions that are in play. In this case, the following are probably relevant:




      Definition: Let ${a_n}$ be a sequence of real numbers. We say that the series
      $$ sum_{n=1}^{infty} a_n $$
      converges if the sequence of partial sums
      $$ S_N := sum_{n=1}^{N} a_n $$
      converges. Oherwise, we say that the series diverges.




      That is, understanding the behaviour of a series comes down to understanding the limiting behaviour of the sequence of partial sums. So, instead of asking about the series, let's step back for a second and ask how the sequence of partial sums might behave. If the series diverges, there are basically three possibilities:




      1. It could be that for any $M > 0$, there exists an $N$ large enough that $n > N$ implies that $S_n > M$. In this case, we say that the sequence of partial sums (and the corresponding series) diverge to $infty$. For example:
        $$ sum_{n=1}^{infty} 1 qquadtext{or}qquad sum_{n=1}^{infty} frac{1}{n}. $$

      2. It could be that for any $M > 0$, there exists an $N$ large enough that $n > N$ implies that $S_n < -M$. In this case, we say that the sequence of partial sums (and the corresponding series) diverge to $-infty$.
        $$ sum_{n=1}^{infty} frac{1}{log(frac{1}{n})}. $$

      3. It could be that neither of these things occurs. In this case, the sequence (and the corresponding series) simply diverge, and no other descriptors are necessary, though we might be able to describe the behaviour in a little more detail if we care to. For example, the partial sums could bounce back and forth between to specific values, or they could oscillate with increasingly large oscillations, or they fall into some cycle, or they could simply bounce around at random.


      In the case of the series presented in the question, the alternating series test shows that the series diverges (as you point out), but this is about the most that we can say without using some other tools. Very roughly speaking, we can see that the difference between consecutive partial sums increases to 1, so the sizes of the oscillations are bounded. There is also enough symmetry in the series to imply that we are not consistently adding more than we are subtracting, hence it is reasonable to think that the sequence of partial sums oscillates, but is bounded.






      share|cite|improve this answer











      $endgroup$
















        0












        0








        0





        $begingroup$

        First, it is very important in mathematics to carefully read and understand the definitions that are in play. In this case, the following are probably relevant:




        Definition: Let ${a_n}$ be a sequence of real numbers. We say that the series
        $$ sum_{n=1}^{infty} a_n $$
        converges if the sequence of partial sums
        $$ S_N := sum_{n=1}^{N} a_n $$
        converges. Oherwise, we say that the series diverges.




        That is, understanding the behaviour of a series comes down to understanding the limiting behaviour of the sequence of partial sums. So, instead of asking about the series, let's step back for a second and ask how the sequence of partial sums might behave. If the series diverges, there are basically three possibilities:




        1. It could be that for any $M > 0$, there exists an $N$ large enough that $n > N$ implies that $S_n > M$. In this case, we say that the sequence of partial sums (and the corresponding series) diverge to $infty$. For example:
          $$ sum_{n=1}^{infty} 1 qquadtext{or}qquad sum_{n=1}^{infty} frac{1}{n}. $$

        2. It could be that for any $M > 0$, there exists an $N$ large enough that $n > N$ implies that $S_n < -M$. In this case, we say that the sequence of partial sums (and the corresponding series) diverge to $-infty$.
          $$ sum_{n=1}^{infty} frac{1}{log(frac{1}{n})}. $$

        3. It could be that neither of these things occurs. In this case, the sequence (and the corresponding series) simply diverge, and no other descriptors are necessary, though we might be able to describe the behaviour in a little more detail if we care to. For example, the partial sums could bounce back and forth between to specific values, or they could oscillate with increasingly large oscillations, or they fall into some cycle, or they could simply bounce around at random.


        In the case of the series presented in the question, the alternating series test shows that the series diverges (as you point out), but this is about the most that we can say without using some other tools. Very roughly speaking, we can see that the difference between consecutive partial sums increases to 1, so the sizes of the oscillations are bounded. There is also enough symmetry in the series to imply that we are not consistently adding more than we are subtracting, hence it is reasonable to think that the sequence of partial sums oscillates, but is bounded.






        share|cite|improve this answer











        $endgroup$



        First, it is very important in mathematics to carefully read and understand the definitions that are in play. In this case, the following are probably relevant:




        Definition: Let ${a_n}$ be a sequence of real numbers. We say that the series
        $$ sum_{n=1}^{infty} a_n $$
        converges if the sequence of partial sums
        $$ S_N := sum_{n=1}^{N} a_n $$
        converges. Oherwise, we say that the series diverges.




        That is, understanding the behaviour of a series comes down to understanding the limiting behaviour of the sequence of partial sums. So, instead of asking about the series, let's step back for a second and ask how the sequence of partial sums might behave. If the series diverges, there are basically three possibilities:




        1. It could be that for any $M > 0$, there exists an $N$ large enough that $n > N$ implies that $S_n > M$. In this case, we say that the sequence of partial sums (and the corresponding series) diverge to $infty$. For example:
          $$ sum_{n=1}^{infty} 1 qquadtext{or}qquad sum_{n=1}^{infty} frac{1}{n}. $$

        2. It could be that for any $M > 0$, there exists an $N$ large enough that $n > N$ implies that $S_n < -M$. In this case, we say that the sequence of partial sums (and the corresponding series) diverge to $-infty$.
          $$ sum_{n=1}^{infty} frac{1}{log(frac{1}{n})}. $$

        3. It could be that neither of these things occurs. In this case, the sequence (and the corresponding series) simply diverge, and no other descriptors are necessary, though we might be able to describe the behaviour in a little more detail if we care to. For example, the partial sums could bounce back and forth between to specific values, or they could oscillate with increasingly large oscillations, or they fall into some cycle, or they could simply bounce around at random.


        In the case of the series presented in the question, the alternating series test shows that the series diverges (as you point out), but this is about the most that we can say without using some other tools. Very roughly speaking, we can see that the difference between consecutive partial sums increases to 1, so the sizes of the oscillations are bounded. There is also enough symmetry in the series to imply that we are not consistently adding more than we are subtracting, hence it is reasonable to think that the sequence of partial sums oscillates, but is bounded.







        share|cite|improve this answer














        share|cite|improve this answer



        share|cite|improve this answer








        edited Jan 1 at 17:34









        Shashi

        7,1731528




        7,1731528










        answered Dec 1 '18 at 18:59









        Xander HendersonXander Henderson

        14.3k103554




        14.3k103554






























            draft saved

            draft discarded




















































            Thanks for contributing an answer to Mathematics Stack Exchange!


            • Please be sure to answer the question. Provide details and share your research!

            But avoid



            • Asking for help, clarification, or responding to other answers.

            • Making statements based on opinion; back them up with references or personal experience.


            Use MathJax to format equations. MathJax reference.


            To learn more, see our tips on writing great answers.




            draft saved


            draft discarded














            StackExchange.ready(
            function () {
            StackExchange.openid.initPostLogin('.new-post-login', 'https%3a%2f%2fmath.stackexchange.com%2fquestions%2f3021606%2falternating-series-doubts%23new-answer', 'question_page');
            }
            );

            Post as a guest















            Required, but never shown





















































            Required, but never shown














            Required, but never shown












            Required, but never shown







            Required, but never shown

































            Required, but never shown














            Required, but never shown












            Required, but never shown







            Required, but never shown







            Popular posts from this blog

            Human spaceflight

            Can not write log (Is /dev/pts mounted?) - openpty in Ubuntu-on-Windows?

            File:DeusFollowingSea.jpg